www.vorhilfe.de
- Förderverein -
Der Förderverein.

Gemeinnütziger Verein zur Finanzierung des Projekts Vorhilfe.de.
Hallo Gast!einloggen | registrieren ]
Startseite · Mitglieder · Impressum
Forenbaum
^ Forenbaum
Status VH e.V.
  Status Vereinsforum

Gezeigt werden alle Foren bis zur Tiefe 2

Navigation
 Startseite...
 Suchen
 Impressum
Das Projekt
Server und Internetanbindung werden durch Spenden finanziert.
Organisiert wird das Projekt von unserem Koordinatorenteam.
Hunderte Mitglieder helfen ehrenamtlich in unseren moderierten Foren.
Anbieter der Seite ist der gemeinnützige Verein "Vorhilfe.de e.V.".
Partnerseiten
Weitere Fächer:

Open Source FunktionenplotterFunkyPlot: Kostenloser und quelloffener Funktionenplotter für Linux und andere Betriebssysteme
Forum "Uni-Analysis-Sonstiges" - Summierbare Familien
Summierbare Familien < Sonstiges < Analysis < Hochschule < Mathe < Vorhilfe
Ansicht: [ geschachtelt ] | ^ Forum "Uni-Analysis-Sonstiges"  | ^^ Alle Foren  | ^ Forenbaum  | Materialien

Summierbare Familien: Frage (beantwortet)
Status: (Frage) beantwortet Status 
Datum: 11:31 So 27.01.2013
Autor: Blackburn4717537

Aufgabe
Sei [mm] \{a_j\}_{j \in J} [/mm] eine Familie nicht-negativer Zahlen und summierbar.
Dann folgt: [mm] \forall \varepsilon [/mm] > 0 [mm] \exists J_0 \in \varepsilon(J): \summe_{j \in J\setminus J_0} [/mm] < [mm] \varepsilon [/mm]

[mm] \varepsilon(J) [/mm] ist die Menge aller endlichen Teilmengen von J.


Hallo,

stimmt die obige Behauptung so?
Wenn ja, wie sieht der Beweis dazu aus?

Grüsse
Alexander

        
Bezug
Summierbare Familien: Antwort
Status: (Antwort) fertig Status 
Datum: 13:13 So 27.01.2013
Autor: Gonozal_IX

Hiho,

> stimmt die obige Behauptung so?
>  Wenn ja, wie sieht der Beweis dazu aus?

wo sind deine eigenen Ansätze dazu?
Dir vorrechnen wird hier sicherlich niemand.

Was heißt es denn, dass eine Familie summierbar ist?
Gilt die Aussage denn für abzählbar viele [mm] a_i [/mm] ?
Und und und.....

MFG,
Gono.

Bezug
        
Bezug
Summierbare Familien: Antwort
Status: (Antwort) fertig Status 
Datum: 14:01 So 27.01.2013
Autor: Helbig


> Sei [mm]\{a_j\}_{j \in J}[/mm] eine Familie nicht-negativer Zahlen
> und summierbar.
>  Dann folgt: [mm]\forall \varepsilon[/mm] > 0 [mm]\exists J_0 \in \varepsilon(J): \summe_{j \in J\setminus J_0}[/mm]

> < [mm]\varepsilon[/mm]
>  
> [mm]\varepsilon(J)[/mm] ist die Menge aller endlichen Teilmengen von
> J.

Hallo Alexander,

Die Aussage stimmt. Aber beim Beweis können wir Dir nur helfen, wenn wir die Definition von "summierbarer Familie" kennen. Von der Schreibweise erinnert das sehr an Königsberger, Analysis I, Abschnitt 6.3. Zeige zunächst, daß für jede endliche Teilmenge [mm] $I\subseteq [/mm] J$ mit [mm] $(a_i\mid i\in [/mm] J)$ auch die Familie [mm] $(a_i\mid i\in J\setminus [/mm] I)$ summierbar ist.

Vielleicht hattet Ihr auch den "Großen Umordnungssatz". Damit ist der Beweis dann sehr kurz.

Dieser Stoff gehört leider nicht zum Kanon typischer Analysiskurse, daher tue ich mich schwer, hier zu helfen.

Übrigens, bezeichne die Menge endlicher Teilmengen von $J$ besser mit [mm] ${\cal E}(J)$ [/mm] statt mit [mm] $\epsilon(J)\,.$ [/mm]

Grüße,
Wolfgang


Bezug
                
Bezug
Summierbare Familien: Mitteilung
Status: (Mitteilung) Reaktion unnötig Status 
Datum: 20:29 Di 29.01.2013
Autor: Blackburn4717537

Hallo Wolfgang und Gonozal_IX,

die Idee war die folgende:

Sei [mm] (a_n) [/mm] eine komplexe Folge und [mm] \summe_{n=0}^{\infty}a_n [/mm] konvergent.
Dann ist die die Folge [mm] (R_n) [/mm] mit [mm] R_{n+1} [/mm] := [mm] \summe_{k=n+1}^{\infty}a_k [/mm] eine Nullfolge (war mal auf einem Übungsblatt). Man lässt ja also die ersten n Glieder der Reihe weg.
Dann hatte ich mir überlegt, dass sich das auch auf summierbare Familien übertragen lassen müsste.

Defintion (Summierbarkeit einer Familie nicht-negativer Zahlen)

Sei J eine beliebige unendliche Indexmenge, und [mm] \{a_j\}_{j \in J} [/mm] eine Familie mit [mm] a_j \ge [/mm] 0 für alle j. Bezeichne mit [mm] {\cal E}(J) [/mm] die Menge aller endlichen Teilmengen von J. Die Familie [mm] \{a_j\}_{j \in J} [/mm] heißt summierbar, falls [mm] \sup\limits_{J_0 \in {\cal E}(J)}\summe_{j \in J_0}a_j [/mm] < [mm] +\infty [/mm]

Meine Behauptung ist jetzt:

Sei [mm] \{a_j\}_{j \in J} [/mm] eine Familie nicht-negativer Zahlen und summierbar.
Dann folgt: [mm] \forall \varepsilon [/mm] > 0 [mm] \exists J_0 \in \varepsilon(J): \summe_{j \in J\setminus J_0} [/mm] < [mm] \varepsilon [/mm]

Der Beweis dazu sieht wie folgt aus (habe ich nicht selber gemacht):

Sei A := [mm] \sup\limits_{J_0 \in {\cal E}(J)}\summe_{j \in J_0}a_j [/mm] < [mm] +\infty. [/mm]
Sei [mm] \varepsilon [/mm] > 0. Dann existiert ein [mm] J_0 \in {\cal E}(J), [/mm] sodass [mm] \summe_{j \in J_0}a_j [/mm] > A - [mm] \varepsilon [/mm] (nach Defintion des Supremums).
[mm] \summe_{j \in J\setminus J_0}a_j [/mm] = [mm] \summe_{j \in J}a_j [/mm] - [mm] \summe_{j \in J_0}a_j [/mm] < A - (A - [mm] \varepsilon) [/mm] = [mm] \varepsilon [/mm]
Da [mm] \varepsilon [/mm] > 0 beliebig ist, folgt die Behauptung.

Gruss
Alexander

Bezug
                        
Bezug
Summierbare Familien: Mitteilung
Status: (Mitteilung) Reaktion unnötig Status 
Datum: 21:02 Di 29.01.2013
Autor: Helbig


> Hallo Wolfgang und Gonozal_IX,
>  
> die Idee war die folgende:
>  
> Sei [mm](a_n)[/mm] eine komplexe Folge und [mm]\summe_{n=0}^{\infty}a_n[/mm]
> konvergent.
>  Dann ist die die Folge [mm](R_n)[/mm] mit [mm]R_{n+1}[/mm] :=
> [mm]\summe_{k=n+1}^{\infty}a_k[/mm] eine Nullfolge (war mal auf
> einem Übungsblatt). Man lässt ja also die ersten n
> Glieder der Reihe weg.
>  Dann hatte ich mir überlegt, dass sich das auch auf
> summierbare Familien übertragen lassen müsste.
>  
> Defintion (Summierbarkeit einer Familie nicht-negativer
> Zahlen)
>  
> Sei J eine beliebige unendliche Indexmenge, und [mm]\{a_j\}_{j \in J}[/mm]
> eine Familie mit [mm]a_j \ge[/mm] 0 für alle j. Bezeichne mit [mm]{\cal E}(J)[/mm]
> die Menge aller endlichen Teilmengen von J. Die Familie
> [mm]\{a_j\}_{j \in J}[/mm] heißt summierbar, falls [mm]\sup\limits_{J_0 \in {\cal E}(J)}\summe_{j \in J_0}a_j[/mm]
> < [mm]+\infty[/mm]
>  
> Meine Behauptung ist jetzt:
>  
> Sei [mm]\{a_j\}_{j \in J}[/mm] eine Familie nicht-negativer Zahlen
> und summierbar.
>  Dann folgt: [mm]\forall \varepsilon[/mm] > 0 [mm]\exists J_0 \in \varepsilon(J): \summe_{j \in J\setminus J_0}[/mm]

> < [mm]\varepsilon[/mm]
>  
> Der Beweis dazu sieht wie folgt aus (habe ich nicht selber
> gemacht):
>  
> Sei A := [mm]\sup\limits_{J_0 \in {\cal E}(J)}\summe_{j \in J_0}a_j[/mm]
> < [mm]+\infty.[/mm]
>  Sei [mm]\varepsilon[/mm] > 0. Dann existiert ein [mm]J_0 \in {\cal E}(J),[/mm]

> sodass [mm]\summe_{j \in J_0}a_j[/mm] > A - [mm]\varepsilon[/mm] (nach
> Defintion des Supremums).
>  [mm]\summe_{j \in J\setminus J_0}a_j[/mm] = [mm]\summe_{j \in J}a_j[/mm] -
> [mm]\summe_{j \in J_0}a_j[/mm] < A - (A - [mm]\varepsilon)[/mm] =
> [mm]\varepsilon[/mm]
>  Da [mm]\varepsilon[/mm] > 0 beliebig ist, folgt die Behauptung.

Hallo Alexander,

Hier benutzt Du

    [mm] $\sum_{j\in J} a_j =\sum_{j\in J_0} a_j [/mm] + [mm] \sum_{j\in J\setminus J_0} a_j\,,$ [/mm]

und dies gilt nach dem Großen Umordnungssatz. Hattet Ihr den?

Gruß,
Wolfgang


Bezug
                                
Bezug
Summierbare Familien: Mitteilung
Status: (Mitteilung) Reaktion unnötig Status 
Datum: 21:15 Di 29.01.2013
Autor: Blackburn4717537

Wir hatten den nicht explizit so genannt, aber geht der so?

Sei [mm] (a_n) [/mm] eine reelle Folge.
Angenommen [mm] \summe_{n=0}^{\infty}a_n^+ [/mm] = [mm] +\infty [/mm] und [mm] \summe_{n=0}^{\infty}a_n^- [/mm] = [mm] +\infty. [/mm] Sei w [mm] \in \IR. [/mm] Dann existiert eine bijektive Abbildung f: [mm] \IN \to \IN, [/mm] sodass [mm] \summe_{n=0}^{\infty}a_{f(n)} [/mm] = w.

[mm] a^{+} [/mm] = [mm] max\{0,a\} [/mm]
[mm] a^{-} [/mm] = [mm] -min\{a,0\} [/mm]

Für Familien hatten wir nichts vergleichbares.

Gruss
Alexander

Bezug
                                        
Bezug
Summierbare Familien: Großer Umordnungssatz
Status: (Mitteilung) Reaktion unnötig Status 
Datum: 22:07 Di 29.01.2013
Autor: Helbig


> Wir hatten den nicht explizit so genannt, aber geht der
> so?
>  
> Sei [mm](a_n)[/mm] eine reelle Folge.
>  Angenommen [mm]\summe_{n=0}^{\infty}a_n^+[/mm] = [mm]+\infty[/mm] und
> [mm]\summe_{n=0}^{\infty}a_n^-[/mm] = [mm]+\infty.[/mm] Sei w [mm]\in \IR.[/mm] Dann
> existiert eine bijektive Abbildung f: [mm]\IN \to \IN,[/mm] sodass
> [mm]\summe_{n=0}^{\infty}a_{f(n)}[/mm] = w.
>  
> [mm]a^{+}[/mm] = [mm]max\{0,a\}[/mm]
> [mm]a^{-}[/mm] = [mm]-min\{a,0\}[/mm]

Nein, der "Große Umordnungssatz" ist größer!

Ist [mm] $J=\bigcup \{J_k\mid k \in K\}$ [/mm] eine disjunkte Vereinigung und die Familie [mm] $(a_i\mid i\in [/mm] J)$ summierbar, so ist jede Teilfamilie [mm] $(a_i\mid i\in J_k)$ [/mm] summierbar und es ist

   [mm] $\sum_{i\in J} a_i [/mm] = [mm] \sum_{k\in K}\sum_{i\in J_k} a_i\,.$ [/mm]

In unserem Fall erhalten wir etwa mit [mm] $K=\{1, 2\}$ [/mm] und [mm] $J_1=J_0, J_2=J\setminus J_0$ [/mm] die Formel

    [mm] $\sum_{i\in J} a_i= \sum_{k=1}^2 \sum_{i\in J_k} a_i [/mm] = [mm] \sum_{i\in J_1}a_i [/mm] + [mm] \sum_{i\in J_2} a_i [/mm] = [mm] \sum_{i\in J_0}a_i [/mm] + [mm] \sum_{i\in J\setminus J_0} a_i\,.$ [/mm]

>  
> Für Familien hatten wir nichts vergleichbares.

Heißt das, Ihr habt die Formel als "offensichtlich" gar nicht begründet?

Gruß,
Wolfgang

Bezug
                                                
Bezug
Summierbare Familien: Mitteilung
Status: (Mitteilung) Reaktion unnötig Status 
Datum: 22:44 Di 29.01.2013
Autor: Blackburn4717537

Okay, das hatten wir doch. Wir haben das ,,Großes Assoziativgesetz" genannt.

Gruss
Alexander

Bezug
Ansicht: [ geschachtelt ] | ^ Forum "Uni-Analysis-Sonstiges"  | ^^ Alle Foren  | ^ Forenbaum  | Materialien


^ Seitenanfang ^
ev.vorhilfe.de
[ Startseite | Mitglieder | Impressum ]